Difference between revisions of "2006 AMC 10A Problems/Problem 1"

 
(Solution)
Line 4: Line 4:
 
<math> \mathrm{(A) \ } 31\qquad \mathrm{(B) \ } 32\qquad \mathrm{(C) \ } 33\qquad \mathrm{(D) \ } 34\qquad \mathrm{(E) \ } 35 </math>
 
<math> \mathrm{(A) \ } 31\qquad \mathrm{(B) \ } 32\qquad \mathrm{(C) \ } 33\qquad \mathrm{(D) \ } 34\qquad \mathrm{(E) \ } 35 </math>
 
== Solution ==
 
== Solution ==
 +
Our answer is simply
 +
<math>(3\cdot 5)+(2\cdot 8)\Longrightarrow 15+16=31,(A)</math>
 +
 
== See Also ==
 
== See Also ==
 
*[[2006 AMC 10A Problems]]
 
*[[2006 AMC 10A Problems]]

Revision as of 14:06, 3 July 2006

Problem

Sandwiches at Joe's Fast Food cost $3 each and sodas cost $2 each. How many dollars will it cost to purchase 5 sandwiches and 8 sodas?

$\mathrm{(A) \ } 31\qquad \mathrm{(B) \ } 32\qquad \mathrm{(C) \ } 33\qquad \mathrm{(D) \ } 34\qquad \mathrm{(E) \ } 35$

Solution

Our answer is simply $(3\cdot 5)+(2\cdot 8)\Longrightarrow 15+16=31,(A)$

See Also